Sie sind auf Seite 1von 30

Solved Paper 2012

All India Engineering Entrance Examination


Instructions
1.

This test consists of 90 questions.

2.

There are three parts in the question paper A, B, C consisting of Physics, Chemistry and Mathematics
having 30 questions in each part of equal weightage. Each question is allotted 4 marks for correct
response.

3.

Candidates will be awarded marks as stated above for correct response of each question. 1/4 mark will
be deducted for indicating incorrect response of each question. No deduction from the total score will
be made if no response is indicated for an item in the answer sheet.

4.

There is only one correct response for each question. Filling up more than one response in any
question will be treated as wrong response and marks for wrong response will be deducted according
as per instructions.

Physics
1. A wooden wheel of

2. The figure shows an experimental plot


discharging of a capacitor in an R-C
circuit. The time constant of this circuit
lies between

(a) 2 SYT
(c) SYT

(c) 50 s and 100 s

(b) SYT
(d) 2SYT

Potential difference
(in volt)

radius R is made of two


R
semi-circular parts (see
figure). The two parts
are held together by a
ring made of a metal
strip of cross-sectional
area S and length L. L is
slightly less than 2R.To fit the ring on the
wheel, it is heated so that its temperature
rises by T and it just steps over the
wheel. As it cools down to surrounding
temperature, it presses the semicircular
parts together. If the coefficient of linear
expansion of the metal is and its
Youngs modulus is Y, the force that one
part of the wheel applies on the other part
is

25
20
15
10
5
0
50

100

150 200

Time (in second)

(a) 150 s and 200 s


(b) 0 and 50 s
(d) 100 s and 150 s

250

300

AIEEE Solved Paper 2012

51

3. In a uniformly charged sphere of total

7. A coil is suspended in a uniform magnetic

charge Q and radius R, the electric field E


is plotted as function of distance from the
centre. The graph which would
correspond to the above will be

field with the plane of the coil parallel to


the magnetic lines of force. When a
current is passed through the coil, it starts
oscillating; it is very difficult to stop. But if
an aluminium plate is placed near to the
coil, it stops. This is due to

(a)

(b)

R r

R r

(c)

(d)

8. The mass of a spaceship is 1000 kg. It is to

R r

R r

4. An electromagnetic wave in vacuum has


the electric and magnetic fields E and B,
which are always perpendicular to each
other. The direction of polarisation is
given by X and that of wave propagation
by k. Then,
(a)
(b)
(c)
(d)

(a) development of air current when the plate is


placed
(b) induction of electrical charge on the plate
(c) shielding of magnetic lines of force as
aluminium is a paramagnetic material
(d) electromagnetic induction in the aluminium
plate giving rise to electromagnetic damping

X || B and X || B E
X || E and k || E B
X || B and k || E B
X || E and k || B E

be launched from the earths surface out


into free space. The value of g and R
(radius of earth) are 10 m/s2 and 6400 km
respectively. The required eneregy for
this work will be
(a) 6.4 1011 J

(b) 6.4 108 J

(c) 6.4 10 J

(d) 6.4 1010 J

9. Helium gas goes through a cycle ABCDA


(consisting of two isochoric and isobaric
lines) as shown in figure. Efficiency of this
cycle is nearly (Assume the gas to be close
to ideal gas)
2p0

5. If a simple pendulum has significant


amplitude (upto a factor of 1/e of original)
only in the period between t = 0 s to t = s,
then may be called the average life of the
pendulum. When the spherical bob of the
pendulum suffers a retardation (due to
viscous drag) proportional to its velocity
with b as the constant of proportionality,
the average life time of the pendulum is
(assuming damping is small) in seconds
(a)

0.693
b

(b) b

(c)

1
b

(d)

2
b

6. Hydrogen atom is excited from ground


state to another state with principal
quantum number equal to 4. Then, the
number of spectral lines in the emission
spectra will be
(a) 2

(b) 3

(c) 5

(d) 6

p0

A
V0

(a) 15.4%
(c) 10.5%

2V0

(b) 9.1%
(d) 12.5%

10. In Youngs double slit experiment, one of


the slit is wider than other, so that
amplitude of the light from one slit is
double of that from other slit. If Im be the
maximum
intensity,
the
resultant
intensity I when they interfere at phase
difference , is given by
Im
9
Im
(c)
5
(a)

(4 + 5 cos )
1 + 4 cos 2

Im
3
Im
(d)
9
(b)

1 + 2 cos 2

2
1 + 8 cos 2

52 JEE Main Solved Papers


11. A liquid in a beaker has temperature (t) at time t

(d)

12. A particle of mass m is at rest at the origin at


time t = 0. It is subjected to a force F(t) = F0 e bt in
the x direction. Its speed v(t) is depicted by
which of the following curves?
F0
mb

F0
mb

(a)

(b)
v(t)

v(t)
t

(c)

t
F0
mb

F0
mb

(d)
v(t)

v(t)
t

maximum height of 10 m. The


maximum horizontal distance that
the boy can throw the same stone
up to will be
(b) 10 m
(d) 20 m

16. This question has Statement I and

loge ( 0)

(c)

loge ( 0)

(b)

15. A boy can throw a stone upto a

(a) 20 2 m
(c) 10 2 m

loge ( 0)

(a)

loge ( 0)

and 0 is temperature of surroundings, then


according to Newtons law of cooling, the correct
graph between log e ( 0 ) and t is

13. Two electric bulbs marked 25 W-220 V and

Statement II Of the four choices


given the statements, choose the
one that describes the two
statements.
Statement I Davisson-Germer
experiment established the wave
nature of electrons.
Statement II If electrons have
wave nature, they can interfere and
show diffraction.
(a) Statement I is false, Statement II is true
(b) Statement I is true, Statement II is false
(c) Statement I is true, Statement II is true;
Statement II is the correct explanation
for Statement I
(d) Statement I is true, Statement II is true;
Statement II is not the correct
explanation of Statement I

17. A thin liquid film formed between a


U-shaped wire and a light slider
supports a weight of 15
. 10 2 N
(see figure). The length of the slider
is 30 cm and its weight negligible.
The surface tension of the liquid
film is

100 W-220 V are connected in series to a 440 V


supply. Which of the bulbs will fuse?
(a) Both

(b) 100 W (c) 25 W

Film

(d) Neither

14. Resistance of a given wire is obtained by


measuring the current flowing in it and the
voltage difference applied across it. If the
percentage errors in the measurement of the
current and the voltage difference are 3% each,
then error in the value of resistance of the wire is
(a) 6%

(b) zero

(c) 1%

(d) 3%

w
1

(a) 0.0125 Nm

(b) 0.1 Nm1


(c) 0.05 Nm1
(d) 0.025 Nm1

AIEEE Solved Paper 2012

53

18. A charge Q is uniformly distributed over

20. A radar has a power of 1 kW and is

the surface of non-conducting disc of


radius R. The disc rotates about an axis
perpendicular to its plane and passing
through its centre with an angular
velocity . As a result of this rotation, a
magnetic field of induction B is obtained
at the centre of the disc. If we keep both
the amount of charge placed on the disc,
and its angular velocity to be constant
and vary the radius of the disc, then the
variation of the magnetic induction at the
centre of the disc will be represented by
the figure

operating at a frequency of 10 GHz. It is


located on a mountain top of height 500 m.
The maximum distance upto which it can
detect object located on the surface of the
earth (Radius of earth = 6.4 106 m ) is

(a)

(b)

(a) 80 km (b) 16 km (c) 40 km (d) 64 km

21. Assume that a neutron breaks into a proton


and an electron. The energy released
during this process is (mass of neutron
of
proton
= 1.6725 10 27 kg, mass
= 1.6725 10 27 kg, mass of electron
= 9 10 31 kg)
(a) 0.73 MeV
(c) 6.30 MeV

(b) 7.10 MeV


(d) 5.4 MeV

22. A Carnot engine, whose efficiency is 40%,


R

(c)

takes in heat from a source maintained at a


temperature of 500 K. It is desired to have
an engine of efficiency 60%. Then, the
intake temperature for the same exhaust
(sink) temperature must be

(d)

19. Truth table for system of four NAND


gates as shown in figure is

23. This question has Statement I and

A
Y
B

(a)

(c)

0
0
1
1

0
1
0
1

0
1
1
0

0
0
1
1

0
1
0
1

0
0
1
1

A
0
0
1
1

B
0
1
0
1

Y
1
1
0
0

A
0
0
1
1

B
0
1
0
1

Y
1
0
0
1

(b)

(d)

(a) Efficiency of Carnot engine cannot be made


larger than 50%
(b) 1200 K
(c) 750 K
(d) 600 K

Statement II. Of the four choices given


after the statements, choose the one that
best describes the two statements.
If two springs S1 and S2 of force constants k1
and k2 , respectively are stretched by the
same force, it is found that more work is
done on spring S1 than on spring S2 .
Statement I If stretched by the same
amount, work done on S1, will be more than
that on S2 .
Statement II k1 < k2
(a) Statement I is false, Statement II is true
(b) Statement I is true, Statement II is false
(c) Statement I is true, Statement II is true;
Statement II is the correct explanation for
Statement I
(d) Statement I is true, Statement II is true;
Statement II is not the correct explanation of
Statement I

54 JEE Main Solved Papers


24. Two cars of masses m1 and m2 are moving
in circles of radii r1 and r2 , respectively.
Their speeds are such that they make
complete circles in the same time t. The
ratio of their centripetal acceleration is
(a) m1r1 : m2 r2
(c) r1 : r2

(b) m1 : m2
(d) 1 : 1

25. A cylindrical tube, open at both ends, has


a fundamental frequency f in air. The
tube is dipped vertically in water so that
half of it is in water. The fundamental
frequency of the air-column is now
(a) f

(b)

f
2

(c)

3f
4

(d) 2f

26. An object 2.4 m in front of a lens forms a


sharp image on a film 12 cm behind the
lens. A glass plate
1 cm thick, of
refractive index 1.50 is interposed
between lens and film with its plane faces
parallel to film. At what distance (from
lens) should object shifted to be in sharp
focus on film?
(a) 7.2 m

(b) 2.4 m (c) 3.2 m

(d) 5.6 m

27. A diatomic molecule is made of two


masses m1 and m2 which are separated by
a distance r. If we calculate its rotational
energy by applying Bohrs rule of angular
momentum quantisation, its energy will
be given by (n is an integer)
(a)
(c)

(m1 + m2 )2 n2 h2
2 m12 m22 r 2
2 n2 h2
(m1 + m2 ) r 2

(b)
(d)

n2 h2
2 (m1 + m2 ) r 2
(m1 + m2 )n2 h2
2 m1m2 r 2

28. A spectrometer gives the following


reading when used to measure the angle
of a prism.
Main scale reading : 58.5 degree
Vernier scale reading : 09 divisions
Given that 1 division on main scale
corresponds to 0.5 degree. Total
divisions on the vernier scale is 30 and
match with 29 divisions of the main scale.

The angle of the prism from the above data


is
(a) 58.59 degree
(c) 58.65 degree

(b) 58.77 degree


(d) 59 degree

29. This question has Statement I and


Statement II. Of the four choices given
after the statements, choose the one that
best describes the two statements.
An insulating solid sphere of radius R has
a uniform positive charge density . As a
result of this uniform charge distribution,
there is a finite value of electric potential
at the centre of the sphere, at the surface of
the sphere and also at a point outside the
sphere. The electric potential at infinite is
zero.
Statement I When a charge q is taken
from the centre of the surface of the sphere
q
its potential energy changes by
.
3 0
The electric field at a
distance r ( r < R) from the centre of the
r
sphere is
.
3 0
Statement II

(a) Statement I is false, Statement II is true


(b) Statement I is true, Statement II is false
(c) Statement I is true, Statement II is true;
Statement II is the correct explanation for
Statement I
(d) Statement I is true, Statement II is true;
Statement II is not the correct explanation of
Statement I

30. Proton, deuteron and alpha particles of


same kinetic energy are moving in
circular trajectories in a constant
magnetic field. The radii of proton,
deuteron and alpha particle are
respectively rp , rd and r . Which one of the
following relation is correct?
(a)
(b)
(c)
(d)

r = rp = rd
r = rp < rd
r > rd > rp
r = rd > rp

AIEEE Solved Paper 2012

55

Chemistry
31. Which among the following will be named

38. The species which can best serve as an

as
dibromidobis
(ethylenediamine)
chromium (III) bromide?

initiator for the cationic polymerisation is

(a) [Cr(en)3 ]Br3

(b) [Cr(en)2 Br2 ]Br

(c) [Cr(en)Br4 ]

(d) [Cr(en)Br2 ]Br

(a) LiAlH4

(b) HNO 3

(c) AlCl 3

(d) BaLi

39. Which of the following on thermal

method of purification is
represented by the following equation?

decomposition yields a basic as well as


acidic oxide?

523 K
Ti(s) + 2I2 (g)
TiI4 (g)

(a) NaNO 3

(b) KClO 3

(c) CaCO 3

(d) NH4NO 3

32. Which

1700 K
Ti(s) + 2I2 (g)

(a) Zone refining


(c) Polling

(b) Cupellation
(d) van-Arkel

33. Lithium

forms body-centred cubic


structure. The length of the side of its unit
cell is 351 pm. Atomic radius of the
lithium will be
(a) 75 pm
(c) 240 pm

(b) 300 pm
(d) 152 pm

Zn2 + / Zn , Ni2 + / Ni and Fe2 + /Fe are


0.76, 0.23 and 0.44 V respectively.
The reaction X + Y 2 + X 2 + + Y will
be spontaneous when
(a) X = Ni, Y = Fe
(c) X = Fe, Y = Zn

(b) X = Ni, Y = Zn
(d) X = Zn, Y = Ni

41. According

34. The molecule having smallest bond angle


is

to Freundlich adsorption
isotherm, which of the following is
correct?

x
p0
m
x
(b)
p1
m
x
(c)
p1/ n
m
(d) All of the above are correct for different range
of pressure
(a)

(b) AsCl 3 (c) SbCl 3

(a) NCl 3

(d) PCl 3

35. Which of the following compounds can be


detected by Molischs test?
(a) Nitro compounds
(c) Amines

(b) Sugars
(d) Primary alcohols

36. The incorrect expression among the


following is
(a)

40. The standard reduction potentials for

Gsystem
= T
Gtotal

(b) In isothermal process,


Wreversible = nRT In
(c) ln K =
(d) K = e

H TS
RT

Vf
Vi

G / RT

37. The density of a solution prepared by


dissolving
120
g
of
urea
(mol. mass = 60 u) in 1000 g of water is
1.15 g/mL. The molarity of this solution is
(a) 0.50 M (b) 1.78 M (c) 1.02 M (d) 2.05 M

42. The equilibrium constant ( Kc ) for the


reaction, N2( g) + O2( g) 2NO( g) at
temperature T is 4 10 4 . The value of Kc
for
the
reaction
1
1
N ( g) + O2( g) at the
NO ( g)
2
2 2
same temperature is
(b) 2.5 102

(a) 0.02
4

(c) 4 10

(d) 50.0

43. The compressibility factor for a real gas at


high pressure is
RT
pb
pb
(c) 1 +
RT

(a) 1 +

(b) 1
(d) 1

pb
RT

56 JEE Main Solved Papers


44. Which one of the following statements is
correct?
(a) All amino acids except lysine are optically
active
(b) All amino acids are optically active
(c) All amino acids except glycine are optically
active
(d) All amino acids except glutamic acid are
optically active

acetyl salicylic acid


phenyl salicylate
acetyl salicylate
methyl salicylic acid

46. Ortho-nitrophenol is less soluble in water


than p and m-nitrophenols because
(a) o-nitrophenol is more volatile steam than those
of m and p-isomers.
(b) o-nitrophenol
shows
intramolecular
H-bonding
(c) o-nitrophenol
shows
intermolecular
H-bonding
(d) melting point of o-nitrophenol is lower than
those of m and p-isomers

47. How many chiral compounds are possible


on monochlorination of 2-methyl butane?
(a) 8
(c) 4

(b) 2
(d) 6

48. Very pure hydrogen (99.9) can be made


by which of the following processes?
(a) Reaction of methane with steam
(b) Mixing natural hydrocarbons of high molecular
weight
(c) Electrolysis of water
(d) Reaction of salts like hydrides with water

49. The electrons identified by quantum


numbers n and l
(1) n = 4, l = 1
(2) n = 4, l = 0
(3) n = 3, l = 2
(4) n = 3, l = 1
can be placed in order of increasing
energy as
(a)
(b)
(c)
(d)

(3) < (4) < (2 ) < (1)


(4) < (2 ) < (3) < (1)
(2 ) < (4) < (1) < (3)
(1) < (3) < (2 ) < (4)

( A) products the concentration of A


changes from 0.1 M to 0.025 M in 40 min.
The rate of reaction when the
concentration of A is 0.01 M is
(a) 1.73 105 M/min (b) 3.47 104 M/min
(c) 3.47 105 M/min (d) 1.73 104 M/min

51. Iron exhibits +2 and +3 oxidation states.


Which of the following statements about
iron is incorrect?

45. Aspirin is known as


(a)
(b)
(c)
(d)

50. For a first order reaction,

(a) Ferrous oxide is more basic in nature than the


ferric oxide
(b) Ferrous compounds are relatively more ionic
than the corresponding ferric compounds
(c) Ferrous compounds are less volatile than the
corresponding ferric compounds
(d) Ferrous compounds are more easily
hydrolysed than the corresponding ferric
compounds

52. The pH of a 0.1 molar solution of the acid


HQ is 3. The value of the ionisation
constant, Ka of the acid is

(a) 3 101

(b) 1 103

(d) 1 107

(c) 1 10

53. Which branched chain isomer of the


hydrocarbon with molecular mass 72 u
gives only one isomer of mono substituted
alkyl halide?
(a)
(b)
(c)
(d)

Tertiary butyl chloride


Neopentane
Isohexane
Neohexane

54. Kf for water is 1.86 K kg mol1. If your


automobile radiator holds 1.0 kg of water,
how many grams of ethylene glycol
(C2H6O2 ) must you add to get the freezing
point of the solution lowered to 2.8C?
(a) 72 g
(c) 39 g

(b) 93 g
(d) 27 g

55. What is DDT among the following?


(a)
(b)
(c)
(d)

Green house gas


A fertiliser
Biodegradable pollutant
Non-biodegradable pollutant

AIEEE Solved Paper 2012


56. The increasing order of the ionic radii of

59. In which of the following pairs, the two

the given isoelectronic species is

species are not isostructural?

(a) Cl ,Ca 2+ ,K + ,S2

(a) CO 2
3 and NO 3

(b) PCl +4 and SiCl 4

(c) PF5 and BrF5

(d) AlF63 and SF6

(b) S ,Cl ,Ca

2+

,K

(c) Ca 2+ ,K + ,Cl ,S2

60. In the given transformation, which of the

(d) K + ,S2 ,Ca 2+ ,Cl

57. 2-hexyne

57

gives
treatment with

trans-2-hexene

(a) Pt /H2
(c) Pd / BaSO 4

(b) Li /NH3
(d) LiAlH4

on

following is the most appropriate reagent?


CH CHCOCH3
Reagent

HO
CH

CHCH2CH3

58. Iodoform can be prepared from all


except
(a)
(b)
(c)
(d)

HO

ethyl methyl ketone


isopropyl alcohol
3-methyl-2-butanone
isobutyl alcohol

s
(a) NH2NH2 , OH

(b) Zn Hg /HCl

(c) Na, Liq. NH3

(d) NaBH4

Mathematics
61. The equation esin x e sin x 4 = 0 has
(a)
(b)
(c)
(d)

1 + (1 + 2 + 4) + (4 + 6 + 9) + (9 + 12 + 16)
+ K + (361 + 380 + 400) is 8000.

infinite number of real roots


no real root
exactly one real root
exactly four real roots

Statement II

62. Let a$ and b$ be two unit vectors. If the


vectors c = a$ + 2 b$ and d = 5a$ 4 b$ are
perpendicular to each other, then the
angle between a$ and b$ is
(a)

(b)

(c)

(d)

63. A spherical balloon is filled with


4500 cu m of helium gas. If a leak in
the balloon causes the gas to escape at
the rate of 72 cu m/min, then the rate
(in m/min) at which the radius of the
balloon decreases 49 min after the
leakage began is
9
7
2
(c)
9
(a)

7
9
9
(d)
2
(b)

64. Statement I The sum of the series

[ k3 ( k 1)3 ] = n3 , for

k =1

any natural number n.


(a) Statement I is false, Statement II is true
(b) Statement I is true, Statement II is true;
Statement II is a correct explanation for
Statement I
(c) Statement I is true, Statement II is true;
Statement II is not a correct explanation for
Statement I
(d) Statement I is true, Statement II is false

65. The negation of the statement If I become


a teacher, then I will open a school, is
(a) I will become a teacher and I will not open a
school
(b) Either I will not become a teacher or I will not
open a school
(c) Neither I will become a teacher nor I will open a
school
(d) I will not become a teacher or I will open a school

58 JEE Main Solved Papers


5 tan x
dx = x
tan x 2
+ a log |sin x 2 cos x | + k,
then a is equal to

66. If the integral

(a) 1
(c) 1

67.

(b) 2
(d) 2

An equation of a common
tangent to the parabola y 2 = 16 3 x and
the ellipse 2 x 2 + y 2 = 4 is y = 2 x + 2 3.
Statement I

4 3
,
m
( m 0) is a common tangent to the
parabola y 2 = 16 3 x and the ellipse
then
m
satisfies
2 x 2 + y 2 = 4,
4
2
m + 2 m = 24.
Statement II

If the line y = mx +

(a) Statement I is false, Statement II is true

69. If

n is a positive
( 3 + 1)2 n ( 3 1)2 n is
(a)
(b)
(c)
(d)

integer,

then

an irrational number
an odd positive integer
an even positive integer
a rational number other than positive integers

70. If 100 times the 100th term of an AP with


non-zero common difference equals the
50 times its 50th term, then the 150th term
of this AP is
(a) 150
(c) 150

(b) 150 times its 50th term


(d) zero

71. In a PQR, if 3 sin P + 4 cos Q = 6 and


4 sin Q + 3 cos P = 1, then the angle R is
equal to
(a)

5
6

(b)

(c)

(d)

3
4

(b) Statement I is true, Statement II is true;


Statement II is a correct explanation for
Statement I

72. A equation of a plane parallel to the plane

(c) Statement I is true, Statement II is true;


Statement II is not a correct explanation for
Statement I

(a) x 2 y + 2 z 3 = 0 (b) x 2 y + 2 z + 1 = 0
(c) x 2 y + 2 z 1 = 0 (d) x 2 y + 2 z + 5 = 0

(d) Statement I is true, Statement II is false

0
0 . If u1 and u2 are

3 2 1
1
column matrices such that Au1 = 0

0
0
and Au2 = 1 , then u1 + u2 is equal to

0

68. Let A = 2

0
1

1
(a) 1

0

1
(b) 1

1

1
(c) 1

0

1
(d) 1

1

x 2 y + 2 z 5 = 0 and at a unit distance


from the origin is

73. If the line 2x + y = k passes through the


point which divides the line segment joining
the points(1, 1) and(2, 4) in the ratio 3 :2, then
k equals
(a)

29
5

(b) 5

(c) 6

(d)

11
5

74. Let x1, x2 , ..., x n be n observations and x be


their arithmetic mean and 2 be the
variance.
Statement I
42 .

Variance of 2 x1, 2 x2 , ..., 2 x n is

Statement II Arithmetic mean


2 x1, 2 x2 , ..., 2 x n is 4x.
(a) Statement I is false, Statement II is true
(b) Statement I is true, Statement II is true; Statement
II is a correct explanation for Statement I
(c) Statement I is true, Statement II is true; Statement
II is not a correct explanation for Statement I
(d) Statement I is true, Statement II is false

AIEEE Solved Paper 2012


75. The population p(t) at time t of a certain
mouse species satisfies the differential
dp(t)
equation
= 0.5(t) 450. If p(0) = 850,
dt
then the time at which the population
becomes zero is
(a) 2 log 18
1
(c) log 18
2

(b) log 9
(d) log 18

given by f( x ) = log | x | + bx 2 + ax , x 0
has extreme values at x = 1 and x = 2.
Statement I f has local maximum at
x = 1 and at x = 2.
1
1
Statement II a = and b =
.
2
4
(a) Statement I is false, Statement II is true
(b) Statement I is true, Statement II is true;
Statement I is a correct explanation for
Statement I
(c) Statement I is true, Statement II is true;
Statement II is not a correct explanation for
Statement I
(d) Statement I is true, Statement II is false

77. The area bounded between the parabolas


y
and x 2 = 9 y and the straight line
4
y = 2 is
x2 =

(b)

10 2
3

(c)

20 2
3

(d) 10 2

78. Assuming the balls to be identical except


for difference in colours, the number of
ways in which one or more balls can be
selected from 10 white, 9 green and
7 black balls, is
(a) 880
(c) 630

x 1 y +1 z 1
and
=
=
2
3
4
x 3 y k z
=
= intersect, then k is equal
1
2
1
to

80. If

the

(a) 1

line

(b)

2
9

(c)

9
2

(d) 0

81. Three numbers are chosen at random

76. Let a, b R be such that the function f

(a) 20 2

59

(b) 629
(d) 879

79. If f : R R is a function defined by


2 x 1
f( x ) = [ x ] cos
, where [ x ] denotes
2
the greatest integer function, then f is
(a) continuous for every real x
(b) discontinuous only at x = 0
(c) discontinuous only at non-zero integral values
of x
(d) continuous only at x = 0

without replacement from {1, 2, 3, ..., 8}.


The probability that their minimum is 3,
given that their maximum is 6, is
(a)

3
8

(b)

1
5

(c)

1
4

(d)

2
5

2
82. If z 1 and z is real, then the point

z 1
represented by the complex number z lies

(a) either on the real axis or on a circle passing


through the origin
(b) on a circle with centre at the origin
(c) either on the real axis or on a circle not passing
through the origin
(d) on the imaginary axis

83. Let P and Q be 3 3 matrices P Q. If


P3 = Q3 and P2Q = Q2 P, then determinant
of ( P2 + Q2 ) is equal to
(a) 2
(c) 0

(b) 1
(d) 1
x

84. If g( x ) = cos 4t dt, then g( x + ) equals


0
g ( x)
g( )
(c) g ( x) g ( )
(a)

(b) g ( x) + g ( )
(d) g ( x) g ( )

85. The length of the diameter of the circle


which touches the X-axis at the point (1, 0)
and passes through the point (2, 3) is
10
3
6
(c)
5

(a)

3
5
5
(d)
3
(b)

86. Let X = {1, 2, 3, 4, 5}. The number of


different ordered pairs ( Y , Z) that can
formed such that Y X , Z X and Y Z
is empty, is
(a) 52
(c) 2 5

(b) 35
(d) 53

60 JEE Main Solved Papers


87. An ellipse is drawn by taking a diameter

of the circle ( x 1)2 + y 2 = 1 as its


semi-minor axis and a diameter of the
circle x 2 + ( y 2)2 = 4 is semi-major axis.
If the centre of the ellipse is at the origin
and its axes are the coordinate axes, then
the equation of the ellipse is
(a) 4 x2 + y2 = 4

(b) x2 + 4 y2 = 8

(c) 4 x2 + y2 = 8

(d) x2 + 4 y2 = 16

89. A line is drawn through the point (1, 2) to


meet the coordinate axes at P and Q such
that it forms a OPQ, where O is the
origin, if the area of the OPQ is least,
then the slope of the line PQ is
(a)

(b) 4
(d)

(c) 2

1
2

90. Let ABCD be a parallelogram such that

88. Consider the function,

AB = q , AD = p and BAD be an acute


angle. If r is the vector that coincides with
the altitude directed from the vertex B to
the side AD, then r is given by

f( x ) = | x 2 | + | x 5 |, x R.

Statement I

1
4

f (4) = 0

Statement II f is continuous in [2, 5],


differentiable in (2, 5) and f(2) = f(5).

(a) r = 3q

(a) Statement I is false, Statement II is true


(b) Statement I is true, Statement II is true;
Statement II is a correct explanation for
Statement I
(c) Statement I is true, Statement II is true;
Statement II is not a correct explanation for
Statement I
(d) Statement I is true, Statement II is false

3(p q)
p
(p p)

p q
(b) r = q +
p
p p
p q
(c) r = q
p
p p
(d) r = 3q +

3(p q)
p
(p p)

Answers
1.
11.
21.
31.
41.
51.
61.
71.
81.

(d)
(a)
(a)
(b)
(d)
(d)
(b)
(b)
(b)

2.
12.
22.
32.
42.
52.
62.
72.
82.

(d)
(c)
(c)
(d)
(d)
(c)
(c)
(a)
(a)

3.
13.
23.
33.
43.
53.
63.
73.
83.

(c)
(c)
(a)
(d)
(c)
(b)
(c)
(c)
(c)

4.
14.
24.
34.
44.
54.
64.
74.
84.

(b)
(a)
(c)
(c)
(c)
(b)
(b)
(d)
(b,c)

5.
15.
25.
35.
45.
55.
65.
75.
85.

(d)
(d)
(a)
(b)
(a)
(d)
(a)
(a)
(a)

6.
16.
26.
36.
46.
56.
66.
76.
86.

(d)
(c)
(d)
(c)
(b)
(c)
(d)
(c)
(b)

7.
17.
27.
37.
47.
57.
67.
77.
87.

(d)
(d)
(d)
(d)
(c)
(b)
(c)
(c)
(d)

8.
18.
28.
38.
48.
58.
68.
78.
88.

(d)
(a)
(c)
(c)
(d)
(d)
(d)
(d)
(c)

9.
19.
29.
39.
49.
59.
69.
79.
89.

(a)
(a)
(a)
(c)
(b)
(c)
(a)
(a)
(c)

10.
20.
30.
40.
50.
60.
70.
80.
90.

(d)
(a)
(b)
(d)
(b)
(a)
(d)
(c)
(b)

Solutions
Physics

6. In emission spectrum, number of bright lines is

1. Elongation due to change in temperature,


l = LT.
Which is compensated by
elastic strain, when temperature
becomes
normal,
F
TL
l =
i .e.,
YS
Q Y = stress = T / A
strain l / L

T
T
TL
Thus,
= LT T = YST
YS
At equilibrium, force exerted by one half on other,
F = 2 T = 2 YST

given by

7. According

to Lenzs law, electromagnetic


induction takes place in the aluminium plate for
which eddy current is developed. This causes loss
in energy which results in damping of oscillatory
motion of the coil.

8. Potential energy on the earths surface is mgR


while in free space, it is zero. So, to free the
spaceship, minimum required energy is
E = mgR
= 103 10 6400 103 J
= 6.4 1010 J

2. Time constant is the duration when the value of


potential drops by 63% of its initial maximum value
(i.e., V0 /e ).
Here, 37% of 25 V = 9.25 V which lies between 100
s to 150 s in the graph.

9. / The efficiency is a measure of degree of


performance i.e., capacity to utilize the heat
into work. In a thermodynamic cyclic process
the area bounded by the cyclic is always gives
the work done during the process. The work
done during different processes may be
negative or positive.

3. Electric field inside the uniformly charged sphere


kQ
r,(r R ), while outside the
R3
sphere, it varies as inverse square of distance,
kQ
E = 2 ; (r R ) which is correctly represented in
r
option (c).

varies linearly, E =

4. In electromagnetic wave, the direction of


propagation of wave, electric field and magnetic
field are mutually perpendicular, i .e., wave
propagates perpendicular to both E and B or
along E B. While polarization of wave takes
place parallel to electric field vector.

Efficiency of a process is defined as the ratio of


work done to energy supplied.
W Area under p - V diagram
Here, =
=
Q
QAB + QBC
where, C p and C V are two heat capacities (molar).
p0 V0
=
nC V T1 + nC p T2
p0 V0
3
5
nR(TB TA ) + nR(TC TD )
2
2
p0 V0
=
3
5
(2 p0 V0 p0 V0 ) + (4 p0 V0 2 p0 V0 )
2
4
p0 V0
=
3
5
p V + 2 p0 V0
2 0 0 2
1
=
= 15.4%
6.5
=

5. For damped harmonic motion,


ma = kx mbv or ma + mbv + kx = 0
Solution to above equation is
bt

k b2
x = A0 e 2 sin t ; with 2 =

4
m
where, amplitude drops exponentially with time.
i .e.,

A = A0 e

b
2

10. Given, a1 = 2 a2

Average time is that duration when amplitude


drops by 63%, i .e., becomes A0 /e.
Thus,
or

A0
= A0 e 2
e
2
b
= 1 or =
b
2

A =

n(n 1) 4(4 1)
=
=6
2
2

I1 = 4I2

Imax = ( I1 +

(as, I A 2 )
I2 )2

(let)
= (3 I2 )2 = 9I2 = 9I0 = Im
(QI1 = I0 , I2 = 4I0 )

62 JEE Main Solved Papers


Now,

I = I1 + I2 + 2 I1I2 cos

15. Maximum speed with which the boy can throw

= 4I0 + I0 + 2 4I0 I0 cos


I
= 5I0 + 4I0 cos = m (5 + 4 cos )
9
I
= m [1 + 4(1 + cos )]
9
I
= m (1 + 8 cos 2 /2 )
9
(Qcos = 2 cos 2 / 2 1)

11. According to Newtons law of cooling, rate of fall in


temperature is proportioal to the difference in
temperature of the body with surrounding, i .e.,
d

= k ( 0 )
dt
d

0 = k dt

ln ( 0 ) = kt + C
which is a straight line with negative slope (i .e., k ).

12. As the force is exponentially decreasing, so its


acceleration i .e., rate of increase of velocity will
decrease with time. Thus, the graph of velocity will be
an increasing curve with decreasing slope with time.
dv F F0 bt
a=
=
=
e
dt
m m
v
t F
0 bt

0 dv = 0 m e dt

with

vmax

F0 1 bt t
F
= 0 e bt

e
mb
m b
0
t
F0
F0
0
bt
bt
=
(e e ) =
(1 e )
mb
mb
F0
=
mb

v=

13. / The power is given by P = VI for ratio value of


current, P is directly proportional to V.
V2
for ratio voltage, the
R
power is inversely proportional to resistance.

It is also given by P =

As the rated power of 25 W is less than 100 W, it


implies that 25 W bulb has higher resistance. As in
series connection, current through both the bulbs is
same but heating in 25 W bulb is more than that of
100 W bulb. So, 25 W bulb will get fused.

14. / For the relation of product type or division type,


the error is counted as the sum of error associated
with each quantity.

From Ohms law,


V
R=
ln R = ln V ln i
i
R V i
=
+
= 3% + 3% = 6%

R
V
i

stone is
u = 2 gh
= 2 10 10
= 10 2 m / s
Range is maximum when projectile is thrown at
an angle of 45. Thus,
u 2 (10 2 )2
Rmax =
=
= 20 m
g
10

16. Davisson

and
Germer
experimentally
established wave nature of electron by
observing diffraction pattern while bombarding
electrons on Ni-crytal.

17. At equilibrium, weight of the given block is


balanced by force due to surface tension, i.e.,
2 LS = W
(where, S = surface + tension and l = 2L,
becauce both surfaces are in air content)
W 1.5 102 N
or
S=
=
2L
2 0.3 m
= 0.025 Nm1

18. / To determine net magnetic field in direction


vector at the centre of the disc, we consider
an elementary ring of thickness dr at a
distance r from the centre and determine the
induction vector due to this elementary part.
Now, for net induction at centre integrate it
within the proper limit.

Taking an elemental ring


of radius r and thickness
dr
dr, we obtain magnetic
R
O
field at the centre of the
r
ring,

dB = 0 di
2r
dq
= 0
2r T
Q
2 r dr
0 R 2
=

2r
2

Q
= 0 2 dr
2 R
So, net magnetic field at the centre of disk,
R
Q R
B = dB = 0 2 dr
0
2 R 0
0Q
=
2 R
0
1
is constant for constant )
i .e., B (as
R
2 R

AIEEE Solved Paper 2012


Energy released = Mass defect c 2

19. The logic circuit is given below

= (mn mp me ) c 2

A.(A.B)

=
A.B

(1.6750 1027 1.6725 1027 9 1031 )


1.66 1027

B
B.(A.B)

Y = (( A ( A B))) (B ( A B))
= ( A + A B) (B + A B)
(using de-Morgans theorem)
= ( A + A B) + (B + ( A B))
= A ( A B) + B ( A B)
(break the line change the sign)
= A ( A + B) + B ( A + B)
= A B + B A = A r B
A

A B

B A

It is the truth table of XOR gate.

16 1031
=
931.5 MeV
1.66 1027
1.6 0.9315
=
MeV 0.9 MeV
1.66
which is close to option (a).
T
22. Efficiency, = 1 sink
Tsource
T
Now,
0.4 = 1 sink
500 K
Tsink = 0.6 500 K
= 300 K
300 K
Thus,
0.6 = 1
T source
300 K
T source =
0.4
= 750 K

23. As no relation between k1 and k2 is given in the


question, that is why, nothing can be predicted
about Statement I. But as in Statement II, k1 < k2
Then, for same force

E
90

ng

Range = (R + h)2 R 2 = 2 Rh + h2
1
~
2 Rh = 2 6400 km
2
= 80 km

21. / According to mass-energy equivalance mass


can assumed to be equivalent of energy and
vice-versa. It is according to Einstein theory of
equivalence.

According to given data, mass of neutron and


proton are equal which do not permit the breaking
up of neutron and proton.
But if we take standard mass of neutron as
1.6750 1027 kg, then
n p+ e

F F2
=
k
k
1
i.e., for constant F
W
k
W1 K 2
=
W2
K1
W = F x = F

distance, for space wave, i.e., line of view).


Ra

931.5 MeV

20. Range of radar on the earth surface (optical


h

63

W1 > W2
But for same displacement
1
1
W = F x = kx x = kx2
2
2
W
K

W K, 1 = 1
W2 K 2
i.e.,
W1 < W2
Thus, in the light of Statement II, Statement I is
false.

24. As their period of revolution is same, so its angular


speed is also same. Centripetal acceleration is
circular path,
a = 2 r.
Thus,

a1 2 r1
r
=
= 1
a2 2 r
r2
2

64 JEE Main Solved Papers


25. Initially for open organ pipe, fundamental
frequency,
o =

v
=f
2l

(given)
=

But when it is half dipped in water, then it becomes


l
closed organ pipe of length . In this case,
2
fundamental frequency,
c =

v
v
v
=
=
=f
l 2l
4l
4
2

present medium of whole set-up, the position of


1
object will be changed by 1 t, where t is

thickness of inserted material medium.

Shift in image position due to glass plate,


1
1
1
S = 1 t = 1
1 cm = cm

1.5
3
For focal length f of the lens,
1
1
1 1 1

= =
f v u 12 240
240
1 20 + 1
or
cm
=
f=
21
f
240
Now, to get back image on the film, lens has to
1
35
form image at 12 cm =
cm such that the

3
3
glass plate will shift the image on the film.
1 1 1
As
=
f v u
1 1 1 3
21
= =

u v f 35 240
48 3 7 21
1
=
=
1680
560
u = 5.6 m

27. / If there is a system of two particles, their centre

n2 h2
n2 h2
=
2 2
8 r
2 r 2
(m1 + m2 )n2 h2
2 m1m2 r 2

30
29
29

1 VSD =
0.5 =
60
30

Thus, least count = 1 MSD 1 VSD


1 29
1
= =
2 60
60
So, reading = main scale reading
+ vernier scale reading
= MSR + n LC
1
= 58.5 + 9
60
= 58.65

29. / A dimensionally incorrect equation must be


incorrect while a dimensionally correct
equation may be correct (it may be differ only in
hammer constant)

Statement I is dimensionally wrong while from


Gauss law,
4
r 3
Q E ds = q
3
E(4 r 2 ) =

0
0

r
E=

30
gives Statement II is correct.

30. For charged particle moving with a speed , in


magnetic field, B, on a circular track of radius
r=

of mass lies on the line joining the two


particles. If the system rotates about a point
(however it may be the centre of mass of two
particles system) it has the rotational kinetic
energy.

Rotational kinetic energy of the two body system


rotating about their centre of mass is
1
RKE = 2 r 2 ,
2
m1m2
where =
= reduced mass
m1 + m2
nh
and angular momentum, L = r 2 =
2
nh
=
2 r 2

where, h = h

28. 1 vernier scale division = 29 main scale division

26. / By inserting any material medium other than

nh 2
1
1
2 r 2 =
r
2
2
2 r 2

RKE =

mv
2 km
=
qB
qB

p2
p = 2 km
Q mv = p and K =
m
2

m
q

or

md = 2 mp
m = 4mp

rp : rd : r =

and qd = q p ;
and q = 2q p

2 mp
4mp
mp
:
:
2q p
qp
qp

= 1: 2 : 1

r = rp < rd

AIEEE Solved Paper 2012

Chemistry
31. / If complex name starts with ligand name or
number, the complex is cationic. The formula of the
cationic complex is written as
[Metal symbol + ligand (with their numbers as
subscript)] + counter ion satisfying the valency of
metal or oxidation of complex.

Two Br, two (en) and one Cr are parts of complex.


2(Br) = 2

Charge on the complex is 2(en) = 0 = + 1


1(Cr) = +3
Thus, complex ion is [Cr(en)2 Br2 ]+ .
Since, anion is bromide, thus complex is
[Cr(en)2 Br2 ]Br.

32. van-Arkel (and de-Boiser) process is based on the


thermal decomposition of a volatile compound like an
iodide, which is first formed by direct combination of
metal to be purified and iodine.
The metallic iodide is then decomposed when heated
at higher temperature and thus the metal obtained is in
the purest form. Titanium and zirconium are purified by
this method.

33. In body-centred cubic structure.


Edge length = a = 351pm, radius = r = ?
3 351
3a
Then,
r=
=
= 152 pm
4
4
34.
N
Group = 15
P

As Valence
=5
Sb electrons
MCl 3 has sp3 hybridised M- element with one
lone-pair.
On moving down the group, the size of the central
atom increases and electronegativity decreases.
Thus, the bond pairs of electrons tend to lie farther
away from the central atom as we move from N to Sb.
Hence, bond pairs repulsion is maximum in NCl 3 and
minimum in SbCl 3 .Thus, bond angle decreases from
NCl 3 (maximum) to SbCl 3 (minimum).
Alternate Solution
According to VSEPR theory, the bond angle increases
with increase in electronegativity of central atom.
Since, electronegativity decreases from N to Sb. Thus,
the order of bond angle in given compounds is
NCl 3 > PCl 3 > AsCl 3 > SbCl 3
Thus, the molecule having smallest bond angle is
SbCl 3 .

65

35. Molischs

test is a general test for


carbohydrates such as glucose fructose,
sucrose, etc. Add two drops of alcoholic
solution of -naphthol to the carbohydrate
solution under study.
Add conc. H2SO 4 slowly along the sides of the
test tube. A violet ring is formed at the junction
of the two liquids which confirms the presence
of a carbohydrate.

36. (a) G = H TS
For a system, total entropy change
= S total
Htotal = 0

Gsystem = TS total
Gsystem
= T

S total
Thus, option (a) is correct.
(b) For isothermal reversible process, E = 0
By first law of thermodynamics,

E = q + W
q = Wreversible
=+

p dV

Work done in isothermal reversible


expansion of n moles of an ideal gas at
temperature T from volume Vi to volume Vf
is
V
W reversible = nRT ln f
Vi
Thus, option (b) is correct.
(c) G = H TS
G = nRT log K = RT log K
for n = 1
RT log K = H TS
H TS
log K =

RT

Thus, option (c) is incorrect.


(d) G = RT log K
G

log K =
RT

K = e G / RT
Thus, option (d) is also correct.

37. / Calculate the volume of solution by the

mass
and moles of
volume
mass
.
solute by the formula, mole =
molar mass
Finally, molarity of solution by the formula,
moles of solute
.
molarity =
volume of solution
formula,

density =

66 JEE Main Solved Papers


Total mass of solution
= 1000 g water + 120 g urea
= 1120 g
Density of solution = 1.15 g / mL
mass
1120 g
Thus, volume of solution =
=
density 1.15 g / mL
= 973.91 mL = 0.974 L
120
Moles of solute =
=2
60
moles of solute
Molarity =
volume (L) of solution
2
=
0.974
= 2.05 mol L1 = 2.05 M

38. Electron-deficient species (Lewis acid) like AlCl 3 ,


BF3 , etc., are is used as an inititator for cationic
polymerisation.

(c) X = Fe, Y = Zn
Fe + Zn2+ Fe 2+ + Zn
Ecell
= EFe
/Fe 2+ + EZn
2+
Ecell
<0
(d) X = Zn, Y = Ni
Zn + Ni 2+ Zn2+ + Ni
Ecell
= E

Zn / Zn 2+

/Ni

41. By Freundlich adsorption isotherm


x
p1
m

x
p0
m

x/m

x
p1/n
m

(c) CaCO 3 CaO + CO 2


Basic

Ni

Ecell
>0

39. (a) 2NaNO 3 2NaNO 2 + O 2

+ E 2+

= 076
. 023
.
= 0.53 V

(b) 2KClO 3 2KCl + 3O 2

/Zn

= 0.44 0.76 = 0.32 V

Acidic

(d) NH4NO 3 N2O + H2O


Pressure ( p)

40. A cell reaction is spontaneous, if G < 0.


Since

G = nFEcell

x
= kp1/ n
m

Thus,

>0
Ecell

x
p1 (in lower pressure range)
m
x
When n is large, = k (independent of pressure)
m
x
Thus,
p0 (in high pressure range
m
when saturation point is reached)
When n = 1,

= Eox
+ Ered

Ecell
red
X + Y 2 + X 2 + + Y
oxd

(a) X = Ni, Y = Fe
Ni + Fe 2 + Ni 2 + + Fe
E

Ni /Ni

Fe 2+ /Fe

Thus,

= 0.44 V

Ecell
= ENi /Ni 2+ + EFe
2+

42. N2 (g ) + O 2 (g ) 2NO(g )
Kc =

= + 0.23 V

2+

[NO]2
= 4 104
[N2 ][O 2 ]

2NO(g ) N2 (g) + O 2 (g)


/Fe

= 0.21 V

Kc =

Ecell
<0
(b) X = Ni, Y = Zn
Ni + Zn2+ Ni 2+ + Zn

Kc =

[N ][O ]
1
1
104
= 2 22 =
=
4
Kc
4
4 10
[NO]
1
1
NO(g )
N (g ) + O 2 (g )
2 2
2
[N2 ]1/ 2 [O 2 ]1/ 2
[NO]

= 0.23 V

= 0.76 V

Kc =

Ecell
= 0.53 V

100
= 50
2

Ni / Ni 2+
Zn 2+ / Zn

(in between low and high pressure)

Ecell
<0

104
4

AIEEE Solved Paper 2012


43. / Compressibility factor for a real gas at high
pressure can be calculated by factor Z =

pv
.
RT

Cl 2
47. CH3 CH2 CH CH3

CH3

van der Waals equation for one mole of real gas is


p + a (V b ) = RT

V2
a
When pressure is high p >> 2 ,
V
p+ a p
such that

V2

H3C CH2 CH CH2 Cl

CH3
(I)

Cl

+ CH3 CH2 C CH3

CH3

p (V b ) = RT
pV pb = RT
pV = RT + pb
Compressibility factor,
pb
pV
z=
= 1 +

RT
RT
Thus,

(II)

Cl

+ CH3 CH CH CH3
*

CH3

44. Glycine is -amino acetic acid with no chiral


carbon, thus optically inactive.

(III)

+ CH2 CH2 CH CH3

CH3
Cl

HCCOOH
NH2 Achiral carbon

(IV)

45. Aspirin is acetyl derivative of salicylic acid.

I and III have chirality (*). Thus, in all four (including


enantiomers) optical isomers are obtained.

OH

48. Hydrides are instant source of hydrogen of higher

COOH

purity. They react with H2O forming H2 gas


CaH2 + 2H2O Ca(OH)2 + 2H2

+ CH3COCl
OCOCH3
COOH

Acetyl salicylic acid

46. There is intramolecular H-bonding in o-nitrophenol


and thus solubility in water is decreased.

+ O

67

Intramolecular H-bonding

49. / Calculate the value of (n + l) , higher the value


of (n + l) , higher the energy. If the value of
(n + l) are same, suborbit with lower the value
of n has lower energy.

(a)
(b)
(c)
(d)

nl (suborbit)

n+l

4
4
3
3

1
0
2
1

4p
4s
3d
3p

5
4
5
4

Higher the value of (n + l ), higher the energy. If


(n + l ) are same, suborbit with lower value of n has
lower energy. Thus,
3 p < 4s < 3d < 4 p
(d) <(b) < (c) < (a)

68 JEE Main Solved Papers


50. By first-order kinetic, rate constant
a
2.303
k=
log

t
a x

a = 0.1 M
(a x) = 0.025 M, t = 40 min
2.303
0.1
k=
log
40
0.025 M
= 0.0347 min1
dx
Rate = = k[ A]1
dt
= 0.0347 0.01
= 3.47 104 M min1

51. (a) FeO > Fe 2O 3 (basic) correct.


(b) FeCl 2 > FeCl 3 (ionic), larger the charge
greater the polarising power, thus greater the
covalent nature. Thus, correct.
(c) Fe 2+ salts are more ionic, thus less volatile
than Fe 3+ salts. Thus, correct.
(d) Greater the covalent nature, more easily they
are hydrolysed. Thus, FeCl 3 is more
hydrolysed than FeCl 2 .
Thus, statement (d) is incorrect.

52. HQ = H+ + Q

10

= K a 0.1

K a = 105

53. Molar mass = 72 = C nH2 n + 2 = 12 n + (2 n + 2 )


n=5
Thus, hydrocarbon is C 5H12 .
Since, it gives only single C 5H11Cl, thus C 5H12 is
symmetrical. It is neo-pentane.
CH3
CH3

H3C C CH3 H3C C CH2Cl

CH3
CH3

54. Coolant is glycol (C 2H6O 2 ) and is non-electrolyte.


Tf = 2.8
1000K f w1
Tf =
m1w2
w1 = 93.33 g

56. For isoelectronic species rn 1


Z

Species

Electrons

Cl
Ca 2 +
K+
S 2

17
20
19
16

18
18
18
18

Thus, ionic size is in order


Ca 2 + < K + < Cl < S2
(b) Li/NH

57. CH3CH2CH2 C C CH3 3


2 -hexyne

H
C == C
CH3CH2CH2

CH3
H

Trans -2 -hexene
(a) Pt /H

2
2-hexyne
n-hexane

2.8 =

cis -2 - hexene

(d) LiAlH

no reaction

58. Iodoform is formed from

C = 0.1 M
103 = K a 0.1
6

was used as an insecticide. DDT is not


metabolised very rapidly by animals. It was also
discovered to have a high toxicity towards fish.
Therefore, it is a non-biodegradable pollutant.

(c) Pd/BaSO

[H+ ] = K aC by Ostwalds dilution law


[H+ ] = 10pH = 103 M
Thus,

55. DDT is P, P Dichlorodiphenyl trichloro ethane. It

1000 1.86 w1
62 1000

(i) CH3CH2OH
(ii) CH3CHO
(iii) All carbonyl compounds of the type
O

R C CH3
(iv) 2 alcohol R CHOH

CH3
O
O

(a) CH3CH2 C CH3 R C CH3


(b) CH3 CH CH3

OH
O

(c) CH3 CH C CH3

CH3
(d) CH3 CH CH2OH

CH3

R CH CH3

OH

Yes
Yes

2 alcohol

R C CH3

Yes

No

AIEEE Solved Paper 2012


59. / The two species are isostructural, if they have
same geometry.

(a) CO 2
3 , NO 3

Tetrahedral
Trigonal bipyramidal
Square pyramidal
Octahedral
CH

CHCOCH3

60. (a)
NH2NH2, OH

glycol

CHCH2CH3

(Wolff-Kishner)

Cl
CHCH2COCH3

Zn(Hg)/HCl

Cl
CH

Na/NH3

sin x

=4

=4

esin

Now, let y = esin x hence, we get


y

CHCOCH3

6a$ b$ = 3
1
a$ b$ =
2

|a$||b$|cos =cos
3

Angle between a$ and a$ is .


3

The rate of decrease of volume of the balloon


is
dV
= 72 m3 /min
dt

61. Given equation is

To find Angle between a and b


Now, c d = 0 (a$ + 2 b$ ) (5a$ 4b$ ) = 0
5a$ a$ 4a$ b$ + 10b$ a$ 8b$ b$ = 0

(ii) Due to a leak, the gas escapes the balloon at


the rate of 72 m3 / min.

Mathematics

sin x

(i) a$ and b$ are unit vectors, i.e.,|a$| = | b$ | = 1


(ii) c = a$ + 2 b$ and d = 5a$ 4b$

(i) Volume (V = 4500 m3 ) of the helium gas


filled in a spherical balloon.

NaBH

5 is not possible for any x R and


5 is also not possible for any x R.

63. Given

4
(d)
low yield

=2 +
=2

NaO

sin x

HCl attacks 2 alcohol as well as (C== C) bond

esin

(iii) c and d are perpendicular to each other,


i.e., c d = 0

HO

sin x

Also, it is obvious that 2 + 5 > e


1
1
and
2 5 < 2 5 , e
e
e

62. Given that


CH

(c)

1
e 1 esin x e esin x , e
e

Hence, we can say that the given equation has no


solution.

HO

(b)

Now, since sine is a bounded function, i.e.,


1 sin x 1, hence we get

Triangular planar

(b) PCl +4 , SiCl 4


(c) PF5
BrF5
(d) AlF63 , SF6

69

1
=4
y

y2 4 y 1 = 0
16 + 4
2
y=2 5

y=

Substituting the value of y, we get


esin x = 2

To find The rate of decrease of the radius of the


balloon 49 min after the leakage started.
dr
i.e.,
@ t = 49 min
dt
(assuming that the leakage started at time t = 0)
Now, since the balloon is spherical in shape,
4
hence the volume of the balloon is V = r 3
3
Differentiating both the sides w.r.t. t, we get
dV 4 2 dr
= 3r
dt
dt
3
dr dV /dt
(i)

=
dt
4 r 2

70 JEE Main Solved Papers


dr
dV
the
@ t = 49 min, we require
dt
dt
radius (r ) at that stage
dV
= 72 m3 / min
dt
Also, amount of volume lost in 49 min
= 72 49 m3 = 3528m3
Now, to find

Now, let S n =

Radius of the balloon at the end of 49 min = 9 m


Hence, from Eq. (i), we get
(dV /dt ) t = 49
dr dV /dt
dr
=

=
2

dt
dt t = 49
4 (r 2 ) t = 49
4 r
dr

dt t

= 49

72
2
= m /min
4 (92 ) 9

64. Statement I
S = (1) + (1 + 2 + 4) + (4 + 6 + 9) + (9 + 12 + 16)
+ K + (361 + 380 + 400)
S = (0 + 0 + 1) + (1 + 2 + 4) + (4 + 6 + 9)
+ (9 + 12 + 16) + K + (361 + 380 + 400)
Now, we can clearly observe the first element in
each bracket.

[r
r =1
n

[k

(r 1)3 ]

(k 1)3 ]

r =1

Substituting the value of k, we get


n

[k

Sn =

Final volume at the end of 49 min


= (4500 3528 ) m3 = 972 m3
If r is the radius at the end of 49 min, then
4 3
r = 972
3

r 3 = 729 r = 9 m

Sn =

(k 1)3 ]

k =1
3

= (1 03 ) + (2 3 13 )
+ (33 2 3 ) + K+ [n3 (n 1)3 ]
Rearranging the terms, we get
S n = 03 + (13 13 ) + (2 3 2 3 ) + (33 33 )
+ K + ((n 1)3 (n 1)3 ) + n3

S n = n3

Since, the number of terms is 20, hence


substituting n = 20, we get
S 20 = 8000
Hence, Statement I is correct.
Statement II We have already proved in the
Statement I), that
Sn =

[k

(k 1)3 ] = n3

k =1

Hence, Statement II is also correct and is a correct


explanation of Statement I.

In third bracket, the first element is 4 = 2 2

Caution If you have solved the series in


Statement I, in the following way, then you may get
option (c) as the correct answer.

In fourth bracket, the first element is 9 = 32

The sum of the n terms of the series is

In second bracket, the first element is 1 = 12

Sn =


In last bracket, the first element is 361 = 192
Hence, we can conclude that there are 20 brackets
in all.
Also, in each of the bracket, there are 3 terms out
of which the first and last terms are perfect squares
of consecutive integers and the middle term is their
product.
The general term of the series is
Tr = (r 1)2 + (r 1) r + (r )2
The sum of the n terms of the series is
Sn =

[(r 1)

+ (r 1) r + (r )2 ]

r =1
n

r 3 (r 1)3
Sn =
r (r 1)
r =1
[Q a3 b 3 = (a b )(a2 + ab + b 2 )]

Sn =

[(r 1)

r =1
n

(3r

+ (r 1) r + (r )2 ]

3r + 1)

r =1

r =1

r =1

Sn = 3 r2 3 r +

r =1

n(n + 1)(2 n + 1)
n(n + 1)

Sn = 3
3
+ n
6
2
Since, the number of terms is 20, hence
substituting n = 20, we get
20 (21)(41)
20(21)
S 20 = 3
3
+ 20
6
2
= 8000
Whenever, we are solving a question of this
Statement I, Statement II type. Then, we should try
our level best to exhaust every possibility that

AIEEE Solved Paper 2012


Statement II cant be used to explain Statement I in
any way before marking (c) as the answer,
otherwise we may end up finding the wrong
answer.
Since, these kind of questions are very tricky and
ambiguous, thats the reason why they are no
more asked in IIT-JEE from the last 2 yr.

65. Let us assume that


p : I become a teacher and
q : I will open a school.
Then, we can easily as certain that
Negation of ( p q ) is ~( p q ) = p ~ q
which means that I will become a teacher and I will
not open a school.
5 tan x
66. Given Integral is
dx
tan x 2
To find The value of a, if
5 tan x
tan x 2 dx = x + a log| sin x 2 cos x| + k(i)
5 tan x
Now, let us assume that I =
dx
tan x 2
Multiplying by cos x in numerator
denominator, we get
5 sin x
dx
I=
sin x 2 cos x
This special integration requires
substitution of type
dDr
Nr = A(Dr ) + B

dx

and

special

Let 5 sin x = A (sin x 2 cos x)


+ B (cos x + 2 sin x)
0 cos x + 5 sin x = ( A + 2 B) sin x
+ (B 2 A) cos x
Comparing the coefficients of sin x and cos x,
we get
A + 2 B = 5 and B 2 A = 0
Solving the above two equations in A and B,
we get
A = 1 and B = 2
5 sin x = (sin x 2 cos x) + 2 (cos x + 2 sin x)
5 sin x
I=
dx
sin x 2 cos x
(sin x 2 cos x) + 2 (cos x + 2 sin x)
dx
=
(sin x 2 cos x)
sin x 2 cos x
(cos x + 2 sin x)
dx
dx + 2
I=
sin x 2 cos x
(sin x 2 cos x)
d (sin x 2 cos x)
I = 1 dx + 2
(sin x 2 cos x)

I = x + 2 log| (sin x 2 cos x)| + k


where, k is the constant of integration.

71
(ii)

Now, by comparing the value of I in Eqs. (i) and (ii),


we get a = 2.

67. Statement I
Given A parabola y2 = 16 3 x and an ellipse
2 x2 + y2 = 4.
To find The equation of common tangent to the
given parabola and the ellipse. This can be very
easily done by comparing the standard equation
of tangents. Standard equation of tangent with
slope m to the parabola y2 = 16 3 x is
4 3
(i)
m
Standard equation of tangent with slope m to the
x2
y2
ellipse
+
= 1 is
2
4
y = mx +

y = mx

2 m2 + 4

(ii)

If a line L is a common tangent to both parabola


and ellipse, then
L should be tangent to parabola, i.e., its equation
should be like Eq. (i).
L should be tangent to ellipse i.e., its equation
should be like Eq. (ii).
i.e., L must be like both of the Eqs. (i) and (ii).
Hence, comparing Eqs. (i) and (ii), we get
4 3
= 2 m2 + 4
m
Squaring, we get
m2 (2 m2 + 4) = 48

m4 + 2 m2 24 = 0

(m2 + 6)(m2 4) = 0

m2 = 4

(Q m2 6)

m= 2

Substituting m = 2 in the Eq. (i), we get the


required equation of the common tangents as
y = 2 x + 2 3 and y = 2 x 2 3
Hence, Statement I is correct.
Statement II In Statement II, we have already
4 3
seen that, if the line y = mx +
is a common
m
2
tangent to the parabola y = 16 3 x and the ellipse
x2
y2
+
= 1, then it satisfies the equation
2
4
4
2
m + 2 m 24 = 0.
Hence, Statement II is also correct but is not able
to explain the Statement I. It is an intermediate step
in the final answer.

72 JEE Main Solved Papers


68. Given Matrices
1
A = 2

0
1
2

0
1
0
0 , Au1 = 0 and Au 2 = 1



1
0
0

1
Adjoint of A = 0

To find Matrix u1 + u 2 .
Since, both Au1 and Au 2 are given, hence
adding them, we get
1 0
Au1 + Au 2 = 0 + 1

0 0
1 + 0
= 0 + 1

0 + 0
1

A(u1 + u 2 ) = 1

0
Since, A is a non-singular matrix, i.e., | A | 0,
hence multiplying both sides by A 1 (from RHS),
we get
1
A 1 A(u1 + u 2 ) = A 1 1

0

1
u1 + u 2 = 2

1
2

| A |= 2
3

1
2

Now,

= 1

1
2

0
0

1
0

1
1

0

(i)

0
1
0
0+ 0
1

Now, cofactor matrix of A (i.e., the matrix in


which every element is replaced by
corresponding cofactor)
2 1
2 0
1 0

2 1
3 2
3 1

1 0
1 0
0 0
=

3 2
3 1
2 1
0 0
1 0
1 0

2 1
2 0
1 0
2
1
0

1
2

1
2

1
0 0
1 0

2 1

1
= 2

1
adj ( A)
A 1 =
| A|
0
1
1

1
A = 2

1 2

0
0

(Q | A | = 1)

Hence, from Eq. (i), we get


1
1 0 0
1

1
u1 + u 2 = 2 1 0

3 2 1
0
1
0
0
1

1 0 1
= 2

1 2 1 0

1+ 0 +
u1 + u 2 = 2 + 1 +

1 2 +

0 1
0 = 1

0 1

69. / Use the formula of binomial expansion

(by expanding the determinant along row 1)


| A| = 1

1
= 0

2
1
0

C1 x n 2 a 2
+ ... + n n Cn a n
n
n
4
n
n 1
and ( x a ) = C0x C1 x
a + n C2 x n 2 a 2
+ ... + ( 1 )n n Cn a n

( x + a )n =

( 3 + 1)2 n =

C0 x n +

C1 x n 1 a +

C 0 ( 3 )2 n +

2n

C1( 3 )2 n 1

2n

C 2 ( 3 )2 n 2 + K +

2n

( 3 1)2 n =

2n

C 0 ( 3 )2 n (1)0 +

2n

C 2 n ( 3 )2 n 2 n

2n

C1( 3 )2 n 1(1)1

2n

+ 2 nC 2 ( 3 )2 n 2 (1)2 + K
C 2 n ( 3 )2 n 2 n (1)2 n

Adding both the binomial expansions above, we get


( 3 + 1)2 n ( 3 1)2 n = 2 [2 n C1( 3 )2 n 1
+

C 3 ( 3 )2 n 3 +

2n

+ K+

C 5 ( 3 )2 n 5

2n

C 2 n 1 ( 3 )2 n ( 2 n 1) ]

2n

which is most certainly an irrational number because


of odd powers of 3 appears in each of the terms.

70. / If first term of an AP is a, common difference is d,


then its nth term will be Tn = a + ( n 1) d.

Given 100 times the 100th term of an


AP = 50 times its 50th term.

AIEEE Solved Paper 2012


To find The 150th term of this AP.
Let a be the first term and d (d 0) be the common
difference of the given AP, then
T100 = a + (100 1)d = a + 99d ,
T50 = a + (50 1)d = a + 49d ,
T150 = a + (150 1)d = a + 149d
Now, according to the question,
100 T100 = 50 T50

100(a + 99d ) = 50(a + 49d )

2(a + 99d ) = (a + 49d )


2 a + 198d = a + 49d
a + 149d = 0
T150 = 0

71. Given A PQR, such that


(i)
3 sin P + 4 cos Q = 6
(ii)
4 sin Q + 3 cos P = 1
Squaring and adding the Eqs. (i) and (ii), we get
(3 sin P + 4 cos Q )2 + (4 sin Q + 3 cos P)2
= 36 + 1
9 (sin2 P + cos 2 P) + 16 (sin2 Q + cos 2 Q )

+ 2 3 4 (sin P cos Q + sin Q cos P) = 37


24[sin (P + Q )] = 37 25
1
sin (P + Q ) =
2

Since, P and Q are angles of PQR, hence 0 < P,


Q < 180

P + Q = 30 or 150

R = 150 or 30 (respectively)
Hence, 2 cases arise here.
Case 1

R = 150
R = 150
P + Q = 30
0 < P, Q < 30
1
sin P < , cos Q < 1
2
3
+ 4
2
11
3 sin P + 4 cos Q <
<6
2
3 sin P + 4 cos Q = 6 is not possible.

Case 2 R = 30
Hence, R = 30 is the only possibility.

72. / We know that perpendicular distance of the


plane ax + by + cz + d = 0 from the point
ax1 + by1 + cz1 + d
a 2 + b2 + c 2

Given A plane P : x 2 y + 2 z 5 = 0
To find The equation of a plane parallel to given
plane P and at a distance of 1 unit from origin.
Equation of family of planes parallel to the given
plane P is
Q : x 2y + 2z + d = 0
Also, perpendicular distance of Q from origin is
1 unit.

0 2(0) + 2(0) + d
12 + 2 2 + 2 2

=1

d
=1
3

d= 3
Hence, the required equation of the plane parallel
to P and at unit distance from origin is
x 2y + 2z 3 = 0
Hence, out of the given equations, option (a) is the
only correct option.

73. Given Line L : 2 x + y = k passes through point


(say P) which divides a line segment (say AB) in
ratio 3 : 2 where A(1, 1) and B(2, 4).
Using section formula, the coordinates of the point
P which divides AB internally in the ratio 3 : 2 are
p

3 2 + 2 1 3 4 + 2 1
8 14
,
P ,
5 5
3+2
3+2

Also, since the line L passes through P, hence


8 14
substituting the coordinates of P , in the
5 5
equation of line L : 2 x + y = k, we get
8
14
2 + = k k = 6
5 5

74. Given x is the AM and 2 is the variance of n


observations x1, x2 , x3 , ..., xn .

3 sin P + 4 cos Q <

(x, y, z,) is d =

73

AM of 2 x1, 2 x2 , 2 x3 , ..., 2 xn
2 x + 2 x2 + 2 x3 + K + 2 xn
= 1
n
x + x2 + x3 + K + xn
= 2 1
= 2x
n

Hence, it is obvious that Statement 2 is false.


From here only we can tell that option (d) is the
correct answer.
However, here we will formally check the validity of
Statement 1.
Variance of 2 x1, 2 x2 , 2 x3 , ..., 2 xn = Variance (2 xi )
= 22
Variance ( xi ) = 4 2

Hence, Statement 1 is correct.


Finally, Statement 1 is true and Statement 2 is
false.

74 JEE Main Solved Papers


75. Given
(i) The population of mouse at time t satisfies
the
differential
equation
dp(t )
p(t ) =
= 0.5 p(t ) 450
dt
(ii) Population of mouse at time t = 0 is
p(0) = 850
To find The time at which the population of the
mouse will become zero, i.e., to find the value of t
at which p(t ) = 0.
Lets solve the differential equation first
dp(t )
= 0.5 p(t ) 450
p(t ) =
dt
2dp(t )
= dt

p(t ) 900
2dp(t )

p(t ) 900 = dt
2 log| p(t ) 900| = t + C, where C is the
constant of integration.
To find the value of C, lets substitute t = 0.

2 log| p(0) 900| = 0 + C

C = 2 log| 850 900|

C = 2 log 50

Solving Eqs. (i) and (ii), we get


1
1
and b =
a=
2
4
x2 + 2
1 1
f ( x) = 2 +
=

2
x
2 x2

f (1) < 0 and f (2 ) < 0


f has local maxima at both x = 1and x = 2 .
Hence, obviously Statement I is correct.
Also, while solving for Statement I, we found the
values of a and b, which justify that Statement II is
also correct.
However, Statement II
Statement I in any way.

t = 2 log 18

76. Given
(i) A
function
f,
such
f( x) = log| x| + bx2 + ax, x 0

that

(ii) The function f has extrema at x = 1 and


i.e.,
and
x = 2,
f (1) = f (2 ) = 0
f (1) 0 f (2 ).
Now, given function f is given by
f( x) = log| x| + bx2 + ax
1
1
f ( x) = + 2 bx + a f ( x) = 2 + 2 b
x
x
Since, f has extrema at x = 1and x = 2.
Hence, f (1) = 0 = f (2 )
(i)
f(1) = 0 a 2 b = 1
1
and
(ii)
f(2 ) = 0 a + 4b =
2

not

explain

77. Given Two parabolas x2 = y and x2 = 9 y


4

To find The area bounded between the parabolas


and the straight line y = 2 .
The required area is equal to the shaded region in
the drawn figure.
Y

Now, substituting the value of C back in the


solution, we get
2 log| p(t ) 900| = t + 2 log 50
Here, since we want to find the value of t at which
p(t ) = 0, hence substituting p(t ) = 0, we get
2 log| 0 900| = t + 2 log 50
900

t = 2 log
50

does

y = 4x 2
1
y = x2
9
y=2

The area of the shaded region (which can be very


easily found by using integration) is twice the area
shaded in first quadrant.
Required area
2
2 5
y

= 2 3 y
y dy
dy = 2 0
0

2
2
y =2

y3 / 2
=5

3 /2 y

=
=0

10 3 / 2
20 2
0) =
(2
3
3

78. Given 10 identical white balls


9 identical green balls, 7 identical black balls
To find The number of ways of select atleast one
ball.
Number of ways to choose zero or more white balls
= (10 + 1) (Qall white balls are mutually identical)
Number of ways to choose zero or more green
balls = (9 + 1)
(Qall green balls are mutually
identical)

AIEEE Solved Paper 2012


Number of ways to choose zero or more black
balls = (7 + 1)
(Qall black balls are mutually
identical)
Hence, number of ways to choose zero or more
balls of any colour = (10 + 1)(9 + 1)(7 + 1)
Also, number of ways to choose a total of zero
balls = 1
Hence, the number, if ways to choose atleast one
ball (irrespective of any colour)
= (10 + 1)(9 + 1)(7 + 1) 1 = 879
NOTE 10 9 7 1 = Number of ways to select atleast
one ball of each colour (in case there is some
confusion)

function f : R R defined by
1
f( x) = [ x] cos x , where [ ] denotes the

79. Given

greatest integer function.


To discuss The continuity of function f.
Now, cos x is continuous, x R
1
cos x is also continuous, x R.

2
Hence, the continuity of f depends upon the
continuity of [ x].
[ x] is discontinuous, x I
So, we should check the conitnuity of f at
x = n, n I.
LHL at x = n is given by
f(n ) = lim f( x)

x n
1
= lim [ x] cos x

2
x n
(2 n 1)
= (n 1) cos
=0
2
RHL at x = n is given by
f(n+ ) = lim f( x)
x n+
1
= lim [ x] cos x

2
x n+

(2 n 1)
=0
2
Also, value of the function at x = n is
1
f(n) = [n] cos n

2
= (n) cos

(2 n 1)
= (n) cos
=0
2
+

Hence, f(n ) = f(n ) = f(n)


f is continuous at x = n, n I

75

Alternate Solution

2x1
f( x) = [ x] cos

= [ x] cos x = [ x] cos x

f( x) = [ x] sin x
Q[ x] sin x is continuous for every real x.

80. / If two lines L1 : x x1 = y y1 = z z1 and


L1 :

a1
b1
c1
x x2
y y2
z z2
intersect, then
=
=
a2
b2
c2

they must have one common point.

x1 y+ 1 z1
=
=
2
3
4
x3 yk z0
and
L2 :
=
=
1
2
1
To find The value of k of the given lines L1 and L2
are intersecting each other.
x1 y+ 1 z1
Let
=
=
= p
L1 :
2
3
4
x3 yk z0
and
=
=
=q
L2 :
1
2
1
Any point P on line L1 is of type

P(2 p + 1, 3 p 1, 4 p + 1) and
any point Q on line L2 is of type
Q(q + 3, 2q + k, q ).
Given Two lines L1 :

Since, L1 and L2 are intersecting each other, hence


both points P and Q should coincide at the point of
intersection, i.e., corresponding coordinates of P
and Q should be same.
2 p + 1 = q + 3, 3 p 1 = 2q + k and 4 p + 1 = q
Solving 2 p + 1 = q + 3 and 4 p + 1 = q , we get the
values of p and q as
3
and q = 5
p=
2
Substituting the values of p and q in the third
equation 3 p 1 = 2q + k,we get
3
3 1 = 2(5) + k
2
9
k=

81. Given 3 numbers are drawn without replacement


from the set {1, 2 , 3, 4, 5, 6, 7, 8}.
To find The probability that their minimum is 3
given that their maximum is 6.
Let A denotes the event that the minimum of the
3 selected numbers is 3 and B denotes the event
that the maximum of the 3 selected numbers is 6,
A
then the required probability is P .
B

76 JEE Main Solved Papers


P ( A B)
A
P =
B
P(B)

Now,

Either ( z z) = 0 or [| z|2 ( z + z)] = 0

P(B) = The probability that the maximum number


selected is 6.
If the maximum number is 6, then
(i) 6 should be one of the 3 selected numbers
(1 way) and

Now, z = z Locus of z is real axis and


[| z|2 ( z + z)] = 0 zz ( z + z) = 0
Locus of z is a circle passing through origin.
Method 2 Put z = x + iy, then
( x2 y2 ) + i (2 xy)
( x + iy)2
z2
=
=
( x 1) + iy
z 1 ( x + iy) 1

(ii) the remaining two numbers should be less


than 6 (i.e., any 2 from 1 to 5) ( 5 C 2 ways)
1 5C 2

10
= 8
C3
C3
Similarly, P( A B) = The probability that the
minimum number is 3 and the maximum
number is 6.

P(B) =

If the minimum number is 3 and the maximum


number is 6, then
(i) 6 should be one of the 3 selected numbers
(1 way).
(ii) 3 should be one of the 3 selected numbers
(1 way) and

( x2 y2 ) + i (2 xy) ( x 1) iy

( x 1) + iy
( x 1) iy

z2
, ( z 1) is purely real, hence its
z1
imaginary part should be equal to zero.

( x2 y2 )( y) + (2 xy)( x 1) = 0
Since,

y( x2 y2 + 2 x 2 x2 ) = 0

y( x2 + y2 2 x) = 0

Either y = 0 or x2 + y2 2 x = 0

Now, y = 0 Locus of z is real axis and

(iii) the remaining 1 number should lie between


3 and 6 (i.e., any one of 4 and 5) (2 ways)

x2 + y2 2 x = 0 Locus of z is a circle passing


through origin.

1 1 2C1

Locus of z is either real axis or a circle


passing through origin.

P( A B) =

C3

P( A B) 2
A
1
Hence, P =
=
=
B
P(B)
10 5
1
Hence, the required probability is .
5

83. Given
(i) Two matrices P and Q of order 3 3 such that
P Q.
(ii) P 3 = Q 3 and P 2Q = Q 2 P

82. Given A complex number

z
, ( z 1) is purely
z1

real.
To find The locus of the complex number z.
z2
, ( z 1) is purely real.
z1

Method 1

Since,

Hence,

z2
z2
=
z1 z1

z2 ( z 1) = z 2 ( z 1)

z2 z z2 = z 2 z z 2

zzz z2 = zzz z 2

z| z|2 z2 = z | z|2 z 2

Rearranging the terms, we get


z| z|2 z | z|2 = z2 z 2

| z|2 ( z z) = ( z z)( z + z)

| z| ( z z) ( z z)( z + z) = 0

( z z) [| z|2 ( z + z)] = 0

To find

The value of determinant of P 2 + Q 2 .

Subtracting the given equations, we get


P 3 P 2Q = Q 3 Q 2 P

P 2 (P Q ) = Q 2 (Q P)

(P Q )(P 2 + Q 2 ) = 0

| P2 + Q 2 | = 0

Reason If A and B are two non-null matrices


such that AB = 0, then both of A and B must be
singular, i.e.,
| A | = | B| = 0.
Proof

Let us assume that A is non-singular, i.e.,


| A | 0.

If| A | 0, then A must be an invertible matrix i.e.,


A 1 must exist uniquely.
Hence, in that case
AB = 0 A 1 AB = A 1 0
(multiplying by A 1 from RHS on both sides)

AIEEE Solved Paper 2012


B = 0 which is a contradiction to the original
condition that A and B are non-null matrices.
Hence, if AB = 0, A, B 0, then A must be singular,
i.e.,| A | = 0.
In similar way, we can prove that matrix B must also
be singular.

Hence, the equation of the circle is


( x h)2 + ( y k )2 = k 2 .
Also, given that the circle passes through points
(1, 0) and (2 , 3). Hence, substituting them in the
equation of the circle, we get
(i)
(1 h)2 + (0 k )2 = k 2

84. Given Integral g( x) = cos 4t dt

(2 h)2 + (3 k )2 = k 2

0 cos 4t d t

g( x + ) =

t = x+

t = 0

cos 4t d t
x+

0 cos 4t d t + x

= g ( x) + I1
I1 =

x+

cos 4t d t =

cos 4t d t
(say)
cos 4t dt

(definite integral property)


= g( )
g ( x + ) = g ( x) + g ( )
But the value of I1 is zero.

sin 4t
sin 4 sin 0
I1 =
=

=0
4
4 0 4

g ( x + ) = g ( x) g ( )
In my opinion, the examiner has made this
question keeping g ( x) + g ( ) as the only answer in
his/her mind. However, he/she did not realise that
the value of the integral I1 is actually zero. Hence, it
does not matter whether you add to or subtract
from g ( x).

85. Given
(i) A circle which touches X-axis at the point (1, 0).
(ii) The circle also passes through the point (2 , 3).
To find The length of the diameter of the circle.
Y

(h , k)
k

(2, 3)

k
O

(1, 0)

(ii)

From Eq. (i), we get h = 1


Substituting in Eq. (ii), we get
(2 1)2 + (3 k )2 = k 2
5

k=
3

To find g ( x + ) in terms of g ( x) and g( ).


g ( x) =

77

Let us assume that the coordinates of the centre of


the circle are C(h, k ) and its radius be r.
Now, since the circle touches X-axis at (1, 0), hence
its radius should be equal to ordinate of centre.

r=k

The diameter of the circle is 2 k =

10
.
3

86. Given A set X = {1, 2, 3, 4, 5}


To find The number of different ordered pairs
(Y , Z ) such that Y X, Z X and Y Z = . Since,
Y X, Z X, hence we can only use the elements
of X to construct sets Y and Z.
Method 1
n(Y )

Number of
ways to make Y

Number of ways to
make Z such that
Y Z=

C1

24

C2

23

C3

22

C4

21

C0

C5

25

20

Let us explain anyone of the above 6 rows say third


row. In third row.
Number of elements in Y = 2
Number of ways to select Y = 5C 2 ways
because any 2 elements of X can be part of Y.
Now, if Y contains any 2 elements, then these
2 elements cannot be used in any way to construct
Z, because we want Y Z = . And from the
remaining 3 elements which are not present in Y,2 3
subsets can be made each of which can be equal
to Z and still Y Z = will be true.
Hence, total number of ways to construct sets Y
and Z such that Y Z =
= 5C 0 2 5 + 5C1 2 5 1 + K + 5C 5 2 5 5
= (2 + 1)5 = 35

78 JEE Main Solved Papers


Method 2 Since, Y X, Z X, hence we can
only use the elements of X to construct sets Y
and Z.
Every elements in X (say a) has four options (as
far as going to Y and Z is concerned).
(1) a Y , i.e., a is present in Y and a Z, i.e.,
a is present in Z.
(2) a Y , i.e., a is present in Y and a Z, i.e.,
a is not present in Z.
(3) a Y , i.e., a is not present in Y and a Z,
i.e., a is present in Z.
(4) a Y , i.e., a is not present in Y and a Z,
i.e., a is not present in Z.
Analysis of the above 4 cases
(1) If a is present in Y and also in Z, then it will
be certainly present in Y Z.
(2) If a is present in Y but not present in Z,
then it will not be present in Y Z.
(3) If a is not present in Y but is present in Z,
then also it will not be present in Y Z.
(4) If a is not present in both of Y and Z, then it
will not be present in Y Z.
We want Y Z = to which only case (a) is not
favourable and remaining cases, i.e., cases
(b), (c) and (d) are favourable.
Hence, for every element a in X, there are
3 favourable ways such that Y Z = .
number
of
ways
Total
= 3 3 3 3 3 = 35 (as there are 5
elements in X and each of them have 3 options
each to go or not to go to Y and Z)
Alternate Solution
Given that
X = {12
, , 3, 4, 5}
and
Y X, Z X, Y Z =
Let a X, then
(i) a Y, a Z
(ii) a Y , a Z
(iii) a Y , a Z
(iv) a Y, a Z
We require Y Z =
Hence, (ii), (iii) and (iv) for a to satisfy
Y Z =
Y Z = has 3 chances for a.
So, for the five elements of X, the number of
required chances is 3 3 3 3 3 3 = 35 .

87. Given
(i) An ellipse whose semi-minor axis coincides with
one of the diameters of the circle ( x 1)2 + y2 = 1.
(ii) The semi-major axis of the ellipse coincides with
one of the diameters of circle x2 + ( y 2 )2 = 4.
(iii) The centre of the ellipse is at origin.
(iv) The axes of the ellipse are coordinate axes.
To find The equation of the ellipse.
Diameter of circle ( x 1)2 + y2 = 1is 2 units and that of
circle x2 + ( y 2 )2 = 4 is 4 units.
Semi-minor axis of ellipse, b = 2 units and
semi-major axis of ellipse, a = 4 units.
Hence, the equation of the ellipse is
x2
y2
x2
y2
+
=1
+ 2 =1
2
16
4
a
b

x2 + 4 y2 = 16

88. Given A function f such that f( x) = | x 2 | + | x 5|


To discuss Continuity and differentiability of f in
interval (2, 5).
x 2, x 2
We know that| x 2 | =
2 x, x < 2
x 5, x 5

and
| x 5| =
5 x, x < 5
2 x, x < 2

| x 2 | = x 2, 2 x 5
x 2, x > 5

5 x, x < 2

and
| x 5| = 5 x, 2 x 5
x 5, x > 5

f( x) = | x 2 | + | x 5|
(2 x) + (5 x), x < 2

= ( x 2 ) + (5 x), 2 x 5 =
( x 2 ) + ( x 5), x > 5

7 2 x, x < 2

2 x 5
3,
2 x 7, x > 5

Now, we can draw the graph of f very easily.


Y
y = 2x 7

y = 7 2x

y=3
O

AIEEE Solved Paper 2012

Now,

4
f(m) = 4 m +

m
4
f (m) = 1 + 2
m
f (m) = 0
m2 = 4

and

m= 2
f(2 ) = 0
f(2 ) = 8

From the graph, we can analyse all the required


things.

Let

Statement I f(4) = 0
It is obviously clear that f is constant around x = 4,
hence, f(4) = 0.
Hence, Statement I is correct.
Statement II
It can be clearly seen that
(i) f is continuous, x [2 , 5]

(ii) f is differentiable, x (2 , 5)
(iii) f(2 ) = f(5) = 3
Hence, Statement II is also correct, but obviously
not a correct explanation of Statement I.

89. Given
(i) A line through (1, 2 ) meets the coordinate axes
at P and Q.
(ii) The area of OPQ is minimum.

79

Since, the area cannot be zero, hence the required


value of m is 2.

90. Given
(i) A parallelogram ABCD such that AB = q and
AD = p.
(ii) The altitude from vertex B to side AD coincides
with a vector r.
To find The vector r in terms of p and q.
Let E be the foot of perpendicular from B to side
AD.
qp
AE = Projection of vector q on p = q p =
| p|

To find
The slope of line PQ.
Q

(1, 2)

Let m be the slope of the line PQ, then the


equation of PQ is
y 2 = m( x 1)
2
Now, PQ meets X-axis at P 1 , 0 and Y-axis at

m
Q(0, 2 m).
2
and OQ = 2 m
OP = 1

m
1
Also, area of OPQ = (OP)(OQ )
2
1
2
=
1 (2 m)
2
m
1
4
=
+2
2 m
2
m
1
4
=
4 m +

2
m

p
A

AE = Vector along AE of length AE


q p
(q p)p
= | AE | AE =
p=
| p|
| p|2
Now, applying triangles law in ABE, we get
AB + BE = AE
(q p)p

q+r =
| p|2
(q p) p

r=

q p
r = q +
p
p p

| p |2

Das könnte Ihnen auch gefallen